¿Por qué las teorías cuánticas de campos relativistas son mucho más restrictivas que las no relativistas?

Parte de la razón por la que la QFT relativista es tan difícil de aprender es que hay montones de 'teoremas de no-go' que descartan ejemplos físicos simples y la intuición física. Una respuesta muy común a la pregunta "¿por qué no podemos hacer X más simple, o pensarlo de esta manera?" es "debido a este teorema de no-go".

Por poner algunos ejemplos, tenemos:

Por supuesto, todos estos teoremas tienen suposiciones adicionales que estoy dejando de lado por brevedad, pero el punto es que la invariancia de Lorentz es una suposición crucial para cada uno.

Por otro lado, la QFT no relativista, tal como se practica en la física de la materia condensada, no tiene tantas restricciones, lo que da como resultado ejemplos mucho mejores. Pero la única diferencia parece ser que trabajan con un grupo de simetría rotacional de S O ( d ) mientras que los físicos de partículas usan el grupo de Lorentz S O ( d 1 , 1 ) , apenas un gran cambio. ¿Existe una razón fundamental e intuitiva por la que la QFT relativista sea mucho más restringida?

Pensé que podría estar relacionado con el hecho de que el grupo de Lorentz no es compacto, pero, de nuevo, la comparación más destacada es en realidad el grupo de Galileo versus el grupo de Poincaré.
Creo que casi todas las cosas que mencionaste son solo una consecuencia de la unitaridad, no necesariamente la invariancia de Lorentz, aunque tal vez en mi pensamiento la invariancia de Lorentz esté entrando de alguna manera sutil. Sin embargo, creo que cosas como Reeh-Schlieder pueden afirmarse de manera no relativista.
Haag no tiene nada que ver con la invariancia relativista: physics.stackexchange.com/q/312389/84967
Hay un teorema de no-go que establece que no se puede comprender intuitivamente por qué la QFT relativista es mucho más restringida.
@Dvij ... que es un corolario del teorema más general de no ir "no puedes entender intuitivamente nada en mecánica cuántica" ;-P
@Dvij ¡Me reiría si no fuera tan cierto!
@knzhou Solo una observación: dado que la QFT no relativista tiene menos restricciones, sospecho que tendrá la libertad de permitir implicaciones adicionales que se descartan en una QFT relativista. Por ejemplo, ¿los bosones de Goldstone son necesariamente partículas de espín 0?

Respuestas (3)

Una de las razones por las que las teorías relativistas son tan restrictivas es por la rigidez del grupo de simetría. Efectivamente, la (parte homogénea) del mismo es simple , frente a la de los sistemas no relativistas, que no lo es.

El grupo de isometría del espacio-tiempo de Minkowski es

PAG o i norte C a r mi ´ = yo S O ( R 1 , d 1 ) = O ( 1 , d 1 ) R d
cuya parte homogénea es O ( 1 , d 1 ) , el llamado Grupo Lorentz 1 . Este grupo es sencillo.

Por otro lado, el grupo de isometría del espacio+tiempo galileano es 2

Bargmann = yo S O ( R 1 × R d 1 ) × tu ( 1 ) = ( O ( d 1 ) R d 1 ) ( tu ( 1 ) × R 1 × R d 1 )
cuya parte homogénea es O ( d 1 ) R d 1 , el llamado (homogéneo) Grupo de Galilei . Este grupo no es semisimple (contiene un subgrupo normal no trivial, el de boosts).

De hecho, existe una clasificación de todos los grupos de simetría cinemática físicamente admisibles (debido a Lévy-Leblond ), que señala a Poincaré como el único grupo con las propiedades anteriores. Existe una sola familia de dichos grupos, que contiene dos parámetros: el radio de AdS y la velocidad de la luz C (y todas las contracciones de İnönü-Wigner invariantes a la rotación de las mismas). Mientras es finito, el grupo es simple. Si lo tomas obtienes Poincaré que tiene un subgrupo normal no trivial, el grupo de traslaciones (y si sacas este grupo por cociente, obtienes un grupo simple, Lorentz). si tambien tomas C obtienes Bargmann (o Galilei), que también tiene un subgrupo normal no trivial (y si sacas el cociente de este grupo, no obtienes un grupo simple; más bien, obtienes Galilei, que tiene un subgrupo normal no trivial, que de refuerzos).

Otra razón es que el postulado de causalidad es trivial en los sistemas no relativistas (porque hay una noción absoluta del tiempo), pero impone fuertes restricciones a los sistemas relativistas (porque no hay una noción absoluta del tiempo). Este postulado se traduce a la teoría cuántica a través del axioma de localidad ,

[ ϕ ( X ) , ϕ ( y ) ] = 0 X , y S t ( X y ) 2 < 0
dónde [ , ] denota un superconmutador. En otras palabras, cualquiera de los dos operadores cuyo soporte se desconecta casualmente debe (super)conmutarse. En los sistemas no relativistas, este axioma es vacío porque todos los intervalos de espacio-tiempo son similares al tiempo, ( X y ) 2 > 0 , es decir, todos los puntos del espacio-tiempo están casualmente conectados. En sistemas relativistas, este axioma es muy fuerte.

Estas dos observaciones se pueden aplicar a los teoremas que cita:

  • Reeh-Schlieder depende del axioma de localidad, por lo que no sorprende que ya no se aplique a los sistemas no relativistas.

  • Coleman-Mandula (ver aquí para una prueba). El grupo de rotación es compacto y por tanto admite representaciones unitarias de dimensión finita. Por otro lado, el grupo de Lorentz es no compacto y por tanto la única representación unitaria de dimensión finita es la trivial. Tenga en cuenta que esto se usa en el paso 4 en la prueba anterior; es aquí donde la prueba se derrumba.

  • Haag también se aplica a sistemas no relativistas, por lo que no es un buen ejemplo del punto de OP. Consulte esta publicación de PSE para obtener más detalles.

  • Weinberg-Witten. Para empezar, este teorema trata sobre partículas sin masa, por lo que no está claro qué significan tales partículas en sistemas no relativistas. Desde el punto de vista de las representaciones irreductibles, pueden tener sentido, al menos en principio. Pero no tienen por qué corresponder a representaciones de helicidades (precisamente porque el pequeño grupo del momento de referencia no es simple). Por lo tanto, el teorema falla (ya que depende crucialmente de las representaciones de helicidad).

  • Spin-estadísticas. Como en Reeh-Schlieder, en los sistemas no relativistas el axioma de localidad es vacuo, por lo que no implica ninguna restricción sobre los operadores.

  • CPT. Ídem.

  • Coleman-Gross. No estoy familiarizado con este resultado, así que no puedo comentar. Ni siquiera sé si se viola en sistemas no relativistas.


1: Más generalmente, el grupo ortogonal indefinido (o pseudo-ortogonal) O ( pag , q ) se define como el conjunto de ( pag + q ) -matrices dimensionales, con coeficientes reales, que dejan invariante la métrica con firma ( pag , q ) :

O ( pag , q ) := { METRO METRO pag + q ( R )     METRO η METRO T η } , η := d i a gramo ( 1 , , 1 pag , + 1 , , + 1 q )

El grupo ortogonal indefinido especial S O ( pag , q ) es el subconjunto de O ( pag , q ) con determinante unitario. Si pag q 0 , el grupo S O ( pag , q ) tiene dos componentes desconectados. En esta respuesta, "grupo de Lorentz" puede referirse al grupo ortogonal con firma ( 1 , d 1 ) ; a su det ( METRO ) + 1 componente; o a su subgrupo ortocrono METRO 0 0 + 1 . Sólo el último es simplemente conexo. La topología del grupo es en su mayoría irrelevante para esta respuesta, por lo que no haremos distinción entre las tres posibles nociones diferentes de "grupo de Lorentz".

2: Se puede probar que el álgebra de Galilei no homogénea, ya diferencia del álgebra de Poincaré, tiene un segundo grupo de cohomología no trivial. En otras palabras, admite una extensión central no trivial. El grupo de Bargmann se define precisamente como el grupo de Galilei no homogéneo extendido centralmente. Estrictamente hablando, todo lo que sabemos es que la extensión central tiene el álgebra R ; a nivel de grupo, podría conducir a un factor de tu ( 1 ) como arriba, o a un factor de R . En mecánica cuántica la primera opción es más natural, porque podemos identificar esta fase con la tu ( 1 ) simetría de la ecuación de Schrödinger (que tiene un grupo de simetría más grande, el llamado grupo de Schrödinger ). Nuevamente, los detalles de la topología del grupo son en su mayoría irrelevantes para esta respuesta.

¿Realmente puedes construir un contraejemplo para Reeh-Schlieder?
@RyanThorngren El concepto de Reeh-Schlieder no relativista no tiene sentido (porque el concepto de álgebras locales tampoco tiene sentido: si C , todos los puntos están causalmente conectados). Por tanto, estrictamente hablando no hay contraejemplos, porque el teorema es incompatible con la situación. Pero en el sentido de OP, que establece "... prohíbe los operadores de posición en QFT relativista", entonces el contraejemplo es sencillo: en sistemas no relativistas, el operador de posición X ^ está bien definida, covariante de Galilei y local. No existe tal operador en el régimen relativista.
Le diría a Reeh-Schlieder que O ( X ) O ( 0 ) 0 , con 0 solo para el operador de identidad. Esto se sigue de la positividad de la reflexión. No sé cómo construir el operador de posición en la mecánica cuántica de muchos cuerpos.
@RyanThorngren 1) La versión de RS que tenía en mente es la de wikipedia , es decir, que el vacío es un vector cíclico (para álgebras sobre conjuntos abiertos en Minkowski). No sé si esto es equivalente a su versión. 2) En principio, solo tomas X ^ = i X ^ i , dónde X ^ i = 1 1 X ^ 1 1 , donde el X ^ esta en la posicion i .
Creo que son equivalentes, o al menos la versión en wikipedia se deriva de la declaración que hice. Los observables locales pueden significar simplemente aquellos cuyo soporte en el espacio de Hilbert está acotado en el espacio real. No creo que el operador X que escribe esté tan acotado en, digamos, un modelo hamiltoniano de salto.
@RyanThorngren Definitivamente no está limitado. Los valores singulares de X puede tomar cualquier valor de a + , por lo que ciertamente es ilimitado. Esto se requiere desde un punto de vista físico: la posición de una partícula puede tomar cualquier valor de a + .

La invariancia de Lorentz también contribuye indirectamente a las restricciones que impone la renormalizabilidad a la teoría. La lógica es algo así:

  1. La acción debe ser invariante de Lorentz, por lo que el número de derivadas espaciales debe ser igual al número de derivadas temporales en la acción.
  2. Queremos que el hamiltoniano desempeñe el papel de la energía (tiene un límite inferior y proporciona estabilidad), por lo tanto, la acción no puede tener más de dos derivadas temporales (para obtener más información, consulte: esta respuesta anterior sobre la renormalización ).
  3. Por 1 y 2, el propagador Δ ( X , y ) necesariamente, divergirá como ( [ X m y m ] [ X m y m ] ) 1 como X y (equivalentemente, la forma del espacio de momento parece ( pag m pag m ) 1 por pag ).

Es ese tercer paso el que conduce a las divergencias que requieren renormalización, y la renormalización es muy restrictiva en cuanto a qué términos se permite que contenga la acción. Sin la invariancia de Lorentz, podríamos agregar más derivadas espaciales sin derivadas temporales, producir un propagador finito de buen comportamiento y trabajar con una clase de teorías mucho más amplia.

Por supuesto, como se discutió en la respuesta vinculada, podría relajarse 2, algo, pero eso no permite ninguna teoría, solo más.

Aquí hay una perspectiva (incompleta), principalmente sobre el infrarrojo:

Para un campo con cargas dadas bajo Lorentz y todas las demás simetrías, existe esencialmente una sola teoría con acción cuadrática de primer orden en derivadas. Para giros enteros es m ϕ m ϕ + metro 2 ϕ 2 y para giros semienteros es ψ ¯ γ m D m ψ + metro ψ ¯ ψ . Este es un hecho de la teoría de la representación, y el hecho de que todo lo que tiene para contraer índices de espacio-tiempo son gramo m v , γ m , y cosas con más índices de espacio-tiempo. Tenga en cuenta que la forma de estas acciones determina que los propagadores desnudos sean los relativistas. 1 / ( pag 2 metro 2 ) y 1 / ( pag metro ) , respectivamente.

Por otro lado, si tuviera que romper la simetría de Lorentz, digamos eligiendo un campo vectorial v m , entonces podrías escribir términos como ϕ v m m ϕ , lo que cambiaría la relación de dispersión para ϕ ser lineal en pag para momentos paralelos a v . Tenga en cuenta que para timelike v esto rompe el grupo de lorentz S O ( 1 , d ) a S O ( d ) .

Para un campo magnético aplicado F i j en fermiones podríamos añadir un término ψ ¯ F i j γ i γ j ψ lo que puede estropear las estadísticas de giro.

Creo que estos nuevos "puntos fijos" gaussianos hacen que muchas cosas se vuelvan torcidas (en el IR) cuando haces teoría de perturbaciones a su alrededor.

Por otro lado, no hay tantos términos que puedan derivar en estos, y por eso la mayoría de las teorías que terminamos estudiando en materia condensada tienen invariancia de Lorentz emergente en la RI. Algunas excepciones significativas son las teorías con una superficie de fermi singular u otras con mezclas ``UV/IR" que hacen que la teoría de campos vea la red.